¿Cómo se cuantifica semiclásicamente el espacio de fase?

A menudo, cuando las personas dan charlas sobre teorías semiclásicas, son muy turbias sobre cómo funciona realmente la cuantización.

Por lo general, comienzan hablando de una partición de -las células luego terminan con algo como la función de onda WKB y poco después hablan sobre el límite 0 .

La cantidad que se cuantifica suele ser la Acción pags d q que se supone que es un medio entero veces 2 π .

¿Cuál es la curva sobre la que integramos? ¿Es una trayectoria, una órbita periódica o algo así? ¿Y cómo se conecta esto a la partición en las celdas de Planck?

Además, ¿cuál es el significado del límite 0 ?

El capítulo 2 de estas notas de clase podría ser útil.
Para una expansión en (más informativo que solo el límite clásico) consulte ns.math.cas.cz/~englis/38.ps
es un número entero veces 2 π , a veces hay un desplazamiento de medio entero. El medio entero proviene del estado fundamental HO y tiene una explicación semiclásica en términos de índices de Maslov, pero la secuencia entera es la regla principal.

Respuestas (3)

Primero, uno debe apreciar que el espacio de fase es clásicamente parametrizado por X , pags y coordenadas en un plano ordinario viajan entre sí, X pags = pags X . Sin embargo, en mecánica cuántica, este no es el caso. En cambio, tenemos el conmutador de Heisenberg

X pags pags X = i .
Esto significa que mecánicamente cuánticamente, el espacio de fase no es un plano ordinario (o un espacio de mayor dimensión) sino una "geometría no conmutativa".

Aún así, también se puede usar el plano ordinario para parametrizar el espacio de fase cuántica. Los operadores en el espacio de Hilbert pueden asociarse canónicamente con funciones ordinarias F ( X , pags ) de las coordenadas de desplazamiento a través de la llamada transformada de Wigner:

http://en.wikipedia.org/wiki/Weyl_ordering

Es importante notar que el conmutador distinto de cero solo afecta la geometría del espacio de fase si lo estudiamos con una mejor resolución que . Para cambios muy grandes de X y pags , el espacio de fase cuántico puede ser aproximado por el clásico.

los 0 es el límite clásico. En este límite, X pags pags X = i también se envía a cero. Entonces, el espacio de fase se convierte efectivamente en una geometría clásica. Si quieres mantener fijo e imagina lo que significa el límite, significa tales valores de X , pags y sus cambios para que Δ X Δ pags . En este límite, el espacio de fase es clásico y se pueden definir integrales de contorno ordinarias en él.

En este límite, el número de "celdas" confinadas en tales contornos es enorme, mucho mayor que uno, norte 1 . Se puede calcular el número de celdas (es decir, el número de vectores base de una base ortonormal que representan una partícula con X , pags en la región bordeada por el contorno) - con una precisión que solo ignora el "límite algo borroso" - tomando el área y dividiéndola por 2 π .

Este mapa se puede visualizar como la siguiente correspondencia entre la traza cuántica y la integral clásica sobre el espacio de fases:

T r 1 2 π d pags d X
Esta relación funciona para cualquier región en el espacio de fase, limitada por cualquier curva. Pero hay clases especiales de curvas para las que podemos decir cosas adicionales.

También podemos dar ejemplos que expliquen por qué funciona la correspondencia anterior. Imagina que el espacio es un círculo de radio R , es decir X se identifica con X + 2 π R , con la circunferencia. En ese caso, el impulso pags está cuantificada porque la función de onda Exp ( i pags X / ) debe tener un solo valor en el círculo. Resulta que

pags = norte R
Los estados propios de pags forman una base completa y podemos contar cuántos estados hay en una región del espacio de fases. Imagine el espacio de fase como una tira delgada (o, más precisamente, un cilindro infinito largo: debido a la identificación periódica de X ). Es delgado en el X dirección, siendo el ancho 2 π R , la circunferencia del círculo, por supuesto.

¿Cuántos estados hay? Si la longitud de la tira/cilindro en el pags la direccion es Δ pags , el número de estados en él es Δ pags R / porque el espaciado es / R . Y debido a que el ancho de la tira es 2 π R , el número de estados por unidad de área del espacio de fase es

Δ pags R 1 Δ pags 1 2 π R = 1 2 π
lo que concuerda exactamente con la afirmación de que 2 π es el área de la celda individual. Este resultado sería cierto no solo para la compactación en el círculo sino también para el infinito X , pags también.

La aproximación WKB utiliza estos conocimientos para derivar muchas otras cosas. En particular, los contornos más interesantes para pags d X en la aproximación WKB son contornos que conectan puntos con H ( X , pags ) = H 0 , es decir, contornos de una energía fija. Estos son lugares del espacio de fase donde se puede imaginar que se localiza un estado propio de energía. Si dibuja muchos contornos de esta forma para varios valores de H 0 y te aseguras de que el área entre ellos sea 2 π para cada par adyacente, puede afirmar que el "anillo" entre contornos representa la región del espacio de fase asociado con un estado propio particular.

Por ejemplo, puede elegir el oscilador armónico. En las unidades correctas elegidas para el gráfico, H ( X , pags ) = H 0 son círculos para cada valor de H 0 y las regiones entre contornos son anillos genuinos. Su ancho será como 1 / norte y su radio será como norte dónde norte es el número entero que etiqueta el nivel de energía. También debe elegir el contorno interno, que rodea el origen, para rodear también el área adecuada del espacio de fase.

El límite clásico solo es lo suficientemente preciso si el espaciado de los contornos es lo suficientemente denso. Sin embargo, aunque es lo suficientemente denso, se puede cuantificar cuál es realmente la densidad, utilizando únicamente los métodos del límite. Para calcular tales cosas, en realidad no estamos usando solo el "límite clásico"; también estamos usando información sobre la "primera corrección cuántica" de la física clásica.

No se deje intimidar, la cuantización semiclásica es muy simple y se puede entender directamente a partir de algunos ejemplos que conducen al caso general.

Considere una partícula en una caja. Los movimientos clásicos son reflejos de la pared. Estos forman una caja en el espacio de fase, ya que la partícula va a la izquierda, golpea la pared, va a la derecha y golpea la otra pared. Si la partícula tiene momento p y la longitud de la caja es L, el área encerrada por este movimiento en el espacio de fase es

pags L

y la condición es que este sea un múltiplo entero de h = 2 π . Esto da la condición de cuantización del momento de la mecánica cuántica.

Para un sistema unidimensional, la regla es que

pags d X = norte h

Con un posible desplazamiento, de modo que el lado derecho podría ser ( norte + 1 / 2 ) h , o ( norte + 3 / 4 ) h , según corresponda, pero el espaciado entre niveles viene dado por esta regla al orden de prioridad en h. Esta regla se puede entender a partir de la relación de deBroglie --- el momento en cualquier x es el número de onda, o la tasa de cambio de la fase de la función de onda. La condición (en unidades naturales donde h = 2 π ) está diciendo que el cambio de fase a medida que sigue una órbita clásica debe ser un múltiplo entero de 2 π , es decir, que la onda debe formar una onda estacionaria.

Esta fórmula no es exacta, porque la onda cuántica no sigue la trayectoria clásica, pero la aproximación WKB solo toma esto como punto de partida, y hace una onda cuya fase viene dada por el valor de esta integral, y cuya amplitud es la recíproco de la raíz cuadrada de la velocidad clásica.

La razón por la que esto funciona ya se conocía antes de que se formulara completamente la teoría cuántica. Pero para entenderlo se requiere familiaridad con las variables de ángulo de acción.

Variables de ángulo de acción

Considere una órbita de una partícula en una dimensión, con posición x y momento p. Llamas al área en el espacio de fase encerrado por la órbita J, y esta es la acción. J es solo una función de H y es constante en el tiempo (por definición).

La variable conjugada a J es una variable que distingue los puntos de la órbita, y se llama θ . Ahora observa que el área en el espacio de fase es invariante bajo transformaciones canónicas (para transformaciones canónicas infinitesimales, este es el teorema de Liouville), de modo que el área entre las órbitas en J y J+dJ es la misma que el área en coordenadas xp entre J y J+dJ, que es simplemente dJ porque esa es la definición de J. Pero esta área en J, θ coordenadas es dJ veces el período de θ , asi que θ tiene el mismo periodo para todo J, que tomaré como 2 π .

La tasa a la que θ aumenta con el tiempo está dada por las ecuaciones de Hamilton

θ ˙ = H j = H ( j )

Y esto es constante en toda la órbita, porque H es constante, y también lo es J. Entonces aprendes que θ aumentan monótonamente a una tasa constante en cada J, y el período de tiempo de θ es:

T = 2 π H ( j )

Cuantificación semiclásica

Suponga que acopla débilmente este sistema unidimensional al electromagnetismo. La frecuencia orbital clásica va a ser la frecuencia de los fotones emitidos (y el doble de esta frecuencia, y tres veces esta frecuencia), por lo que si desea tener transiciones discretas de emisión de fotones, debe asegurarse de emitir un fotón de frecuencia F = 1 T , y quitando energía h F te deja con un estado cuántico al que caer. Entonces, si hay un estado cuántico correspondiente a un movimiento clásico con un valor de J, con energía H(J), debe haber otro estado cuántico con energía

H ( j ) 2 π h T = H ( j ) H ( j ) h H ( j h )

en otras palabras, los estados cuánticos deben estar espaciados uniformemente en J. En este orden, esto significa que hay estados en Jh, J-2h, J-3h, etc., y las transiciones a estos estados tienen que reproducir los armónicos de radiación clásicos. producido cuando acoplas débilmente la cosa al electromagnetismo.

Entonces la regla de cuantización es j = norte h , hasta un posible desplazamiento. La derivación deja en claro que solo es cierto para el orden principal en h. Este fue el argumento de correspondencia de Bohr para la condición de cuantización.

Cuando tiene más de un grado de libertad y el sistema es integrable, tiene variables de acción j 1 , j 2 . . . j norte y variables angulares conjugadas periódicas con período 2 π cada. Puede acoplar débilmente cualquiera de los grados de libertad al electromagnetismo, y cada período clásico del θ variable en el tiempo es

T k = H j k

por lo que el enunciado es que para cada órbita, cada variable J se cuantifica de acuerdo con la regla de Bohr.

j k = norte h

los j k variable es el área encerrada en la proyección unidimensional del movimiento en aquellas coordenadas donde el movimiento se separa en un movimiento multiperiódico (este es el toroide de la respuesta de Bar Moshe). Esta es la extensión de Sommerfeld de la cuantización de Bohr.

Entonces la integral pags d q se toma con p y q cualquier variable conjugada que realice un movimiento de período. En 1d, no hay nada que hacer, en múltiples dimensiones, solo elige variables que ejecutan por separado un movimiento 1d y, en general, debe encontrar J variables. Este procedimiento no funciona para sistemas caóticos clásicos.

Gran respuesta @Ron. ¿Puedes explicar por qué hay un 2 π factor en la primera ecuación de la sección "Cuantización semiclásica"?

Muchas variedades simplécticas (espacios de fase de sistemas mecánicos) admiten un sistema de coordenadas donde la forma simpléctica dos puede escribirse localmente como:

ω = i d pags i d q i + j d yo j d θ j

Dónde pags i , q i son coordenadas lineales yo j son coordenadas radiales y θ j son coordenadas angulares.

La subvariedad parametrizada por θ j es un toro y un resultado debido a que Snyatycki establece que el sistema se puede cuantificar mediante un espacio de Hilbert de funciones de onda (distribuciones) apoyado solo en puntos cuyas coordenadas yo j satisfacer:

2 yo j = metro

Uno de los ejemplos más simples que admiten tal cuantización es el de dos esferas cuya forma simpléctica es su forma de área

A = r s i norte θ d θ d ϕ = d z d ϕ

dónde θ , ϕ son las coordenadas esféricas y z es la coordenada a lo largo del eje.

En este caso la condición de Bohr-Sommerfeld viene dada por:

z = metro 2 ,

que es la condición de cuantificación de proyección de espín.

En un lenguaje matemático más sofisticado se dice que un subconjunto denso abierto de la variedad simpléctica está foliado por toros lagrangianos y la integración es sobre los ciclos generadores de los toros.

Gracias por la respuesta. Desafortunadamente, tendré que masticarlo durante bastante tiempo para entenderlo, ya que no tengo idea de qué es la forma simpléctica de dos y qué hace el producto del sombrero. Además, esta es la primera vez que escucho sobre los estados de Snyatycki.
@ user9886: la estructura simpléctica del paquete cotangente (es decir, espacio de momento) es responsable de la en las ecuaciones de Hamilton (y también en los corchetes de Poisson); sin embargo, las ecuaciones de Hamilton solo toman su forma simple si usa coordenadas canónicas; si permite transformaciones de coordenadas arbitrarias, obtendrá coeficientes más complejos, en particular, los componentes de un tensor de rango 2 antisimétrico, la forma simpléctica; de hecho, una forma simpléctica en una variedad arbitraria es suficiente para hacer la mecánica hamiltoniana (por ejemplo, puede olvidarse de la estructura de paquetes de vectores)